Mathematical and Physical Journal
for High Schools
Issued by the MATFUND Foundation
Already signed up?
New to KöMaL?

Problem B. 4040. (November 2007)

B. 4040. a, b, c are positive real numbers, and ab+bc+ca=1. Prove that


\frac{1-a^2}{1+a^2}+\frac{1-b^2}{1+b^2}+\frac{1-c^2}{1+c^2}\le\frac{3}{2}.

(5 pont)

Deadline expired on December 17, 2007.


Sorry, the solution is available only in Hungarian. Google translation

Megoldás: A nevezőkkel való beszorzás és rendezés után a bizonyítandó állítás a

9a2b2c2+5(a2b2+b2c2+c2a2)+(a2+b2+c2)-3\ge0

alakot ölti. A feltételt felhasználva

a2+b2+c2=(a+b+c)2-2(ab+bc+ca)=(a+b+c)2-2

és

a2b2+b2c2+c2a2=(ab+bc+ca)2-2(ab2c+bc2a+ca2b)=1-2abc(a+b+c),

vagyis a

(3abc)2-10abc(a+b+c)+(a+b+c)2\ge0

egyenlőtlenséget kell igazolnunk. A baloldalt átírhatjuk

\Bigl( 3abc-\frac{1}{3}(a+b+c)\Bigr)^2+\frac{8}{3\sqrt{3}}(a+b+c)
\Bigl(\frac{a+b+c}{\sqrt{3}}-3\sqrt{3}abc\Bigr)

alakba. Az x=\sqrt{3}a, y=\sqrt{3}b, z=\sqrt{3}c helyettesítéssel tehát csak az

\frac{x+y+z}{{3}}\ge xyz

egyenlőtlenséget kell igazolnunk. A feltételt xy+yz+zx=3 alakra átírva a jól ismert x2+y2+z2\gexy+yz+zx egyenlőtlenség miatt

\Bigl(\frac{x+y+z}{{3}}\Bigr)^2=\frac{(x^2+y^2+z^2)+2(xy+yz+zx)}{9}\ge 1,

a számtani-mértani közepek között fennálló összefüggés miatt pedig

1=\frac{xy+yz+zx}{{3}}\ge{\root{3}\of{x^2y^2z^2}}=(xyz)^{2/3}

adódik, ahonnan valóban

\frac{x+y+z}{{3}}\ge 1\ge xyz.

A megoldásból az is látszik, hogy egyenlőség pontosan az a=b=c=1/\sqrt{3} esetben áll fenn.


Statistics:

72 students sent a solution.
5 points:55 students.
4 points:3 students.
3 points:1 student.
2 points:1 student.
1 point:5 students.
0 point:7 students.

Problems in Mathematics of KöMaL, November 2007